プロが教えるわが家の防犯対策術!

14の2おしえてください
数学2です

「14の2おしえてください 数学2です」の質問画像

A 回答 (2件)

有名な解き方は二つあります.



一つは,数学1の2次関数の単元で学んだような,グラフを活用して解く方法.
すなわち,グラフを考察することで得られる不等式
 ・2次方程式の判別式
 ・放物線の軸の位置に関する不等式
 ・ある x での y 座標に関する不等式
を連立して m の値の範囲を求める,という方法です.
(覚えていらっしゃるでしょうか?)

ただ,今回は小問(1)のような誘導もありますし,出題者が期待しているのはもう一つの方法でしょう.
そちらについて説明しますね.

ここでのポイントは,“2次方程式の解と係数の関係”を活用することでできるだけ計算を楽に済ませる,ということで,
そのために条件の表し方を工夫します.

今回の問題はひとまず置いておいて,例えば,
「α, β がともに負」という条件が
 αβ > 0 …(*) かつ α + β < 0 …(**)
 (ただし α, β は実数)
と表せることはよいでしょうか?
「α, β がともに負」ならば(*)も(**)も成り立ちますね.
また,逆に,(*)と(**)が成り立てば,
(*)から
 「α, β がともに正」または「α, β がともに負」
であるとわかり,
そこに(**)を加味することで,「α, β がともに負」に可能性が絞り込まれますね.
したがって,「α, β がともに負」という条件と「(*)かつ(**)」という条件は同値なのです.

ここで,不等式(*), (**)をよく見ると,これはまさに解と係数の関係が使える状況です.
このような形で条件を表すことにより計算を楽に済ませようとしていくわけです.

さて,今回の問題に戻りましょう.
今回の問題で与えられている条件「α, β がともに 3 より小さい」というのは,
言い換えると「(α - 3), (β - 3) がともに負」ということですね.
(小問(1)で α - 3, β - 3 が登場していることが大ヒント.ただ,このヒントがなくても気付いてほしいところです.)
そのため,上の例と同様に考えて,条件を
 (α - 3) (β - 3) > 0 かつ (α - 3) + (β - 3) < 0
 (ただし α, β は実数)
と表します.
あとはここに解と係数の関係を用いていけばよいのです.
その際,元の2次方程式ではなく,小問(1)で求めた2次方程式に対する解と係数の関係を用いれば,計算を少し省略できます.
「α, β が実数」という条件に関しては,元の2次方程式の判別式で処理できますね.
    • good
    • 0

(解法1)


f(x) =x^2+mx+1 とすると、f(x)の2つの解αとβがともに3より小さくなるには、
① f(x)が2つの解をもつ ※問題文で「異なる2つの解」と言ってないので、重解も含める。
→ f(x)=0の判別式 D = m^2-4 =(m+2)(m-2)≧0 よって、m≦-2, 2≦m … (A)

② f(x) の2つの解がともに3より小さい
→ (y=f(x) は「下に凸な放物線」なので、)
f(3) = 3m+10 > 0 よって m>-10/3 … (B)

(A)~(B)を合わせて、-10/3 <m≦-2, 2≦m (答)

(解法2)
f(x) =x^2+mx+1 とすると、
x^2+mx+1 = 0 より、mx = -x^2-1
g(x) =-x^2-1 、h(x) = mxとすると、
2つのグラフ y=g(x) と y=h(x) の交点のx座標が、
f(x) =0の2つの解 α、βである。

2つのグラフ
y=g(x) … y=-x^2 をy軸方向に-1だけ移動した放物線
と y=h(x) … 原点を通る、傾きmの直線
を書いてみてmの値を変えていくと、交点の数が変わっていくのが分かる。

y=g(x) と y=h(x) が接するのは、
f(x)=0の判別式 D = m^2-4 =(m+2)(m-2)=0 よって、m=±2 … (A)
m=2のとき、h(x) =2x。 f(x)=0 の解は x=-1(重解)< 3.(題意を満たす)
m=-2のとき、h(x) =-2x。 f(x)=0 の解は x=1(重解)< 3.(題意を満たす)

y=h(x)の傾き mについて、m<-2, 2<m … (A)' のとき、
交点を2つ持つが、 そのx座標 α、βは ともに3より小さくないといけないので、
x=3 で 「 h(3) > g(3) (y=h(x)のグラフがy=g(x)より上側にある)」 である必要がある。
h(3)-g(3) = f(3) >0 よって m>-10/3 … (B)

(A)(A)'(B)を合わせて、-10/3 <m≦-2, 2≦m (答)
    • good
    • 0

お探しのQ&Aが見つからない時は、教えて!gooで質問しましょう!